A bed frame normally weighs 60 pounds. A new,
stronger material increases the durability of the
frame but makes it weigh 30% more. How many
pounds does the new frame weigh?
F. 63
G. 66
H. 78
J. 90
K. 102

Answers

Answer 1
(H) 78 is how much the new frame should weigh.

Related Questions

Given that triangle ABC ~ XYZ, AB= 4,XY=3,and XZ= 5, find AC

Given that triangle ABC ~ XYZ, AB= 4,XY=3,and XZ= 5, find AC

Answers

Answer:

\(AC = \frac{20}{3}\)

Step-by-step explanation:

Given

\(AB = 4\)

\(XY = 3\)

\(XZ = 5\)

Required

Find AC

ABC XYZ

To solve AC, we make use of the following equivalent ratios.

\(AB : AC = XY : XZ\)

This gives:

\(4 : AC = 3 : 5\)

Express as fraction

\(\frac{4 }{ AC }= \frac{3 }{ 5}\)

Cross multiply

\(3 * AC = 4 * 5\)

\(3 * AC = 20\)

Solve for AC

\(AC = \frac{20}{3}\)

Decide what values of the variable cannot be solutions for each equation.
\(\frac{1}{4x} - \frac{2}{x} = 3\)

Answers

Step-by-step explanation:

\( \frac{1}{4x} - \frac{2}{x} = 3 \\ \frac{1}{4x} - \frac{2}{x} \times \frac{4}{4} = 3 \\ \frac{1}{4x} - \frac{8}{4x} = 3 \\ \frac{1 - 8}{4x} = 3 \\ \frac{ - 7}{4x} = 3 \\ 4x = \frac{ - 7}{3} \\ x = \frac{ - 7}{12} \\ thank \: you\)

Answer:

\(→ \frac{1}{4x} - \frac{2}{x} = 3 \\ \frac{1}{4x} - \frac{(2 \times 4)}{(x \times 4)} = 3 \\ \frac{1}{4x} - \frac{8}{4x} = 3 \\ \frac{(1 - 8)}{4x} = 3 \\ \frac{( - 7)}{4x} = 3 \\ \frac{1}{x}= \frac{(3 \times 4)}{( - 7)} \\ \frac{1}{x}= \frac{12}{( - 7)} \\ \boxed{x = \frac{( - 7)}{12} }\)

x = -7/12 is the right answer.

use the laws of logarithm to expand the term log (ab)^2; log calculator; what is the answer to this math problem; desmos; desmos calculator; wolfram alpha; how to solve log x; how to solve for exponent x

Answers

The expanded form of log \((ab)^{2}\)  using the laws of the logarithm is 2log a + 2log b.

The logarithm is the exponent or power to which a base must be raised to yield a given number. Expressed mathematically, x is the logarithm of n to the base b if \(b^x\) = n, which can be written as x = \(log_{b}\) n.

For example,  \(3^{2}\) = 9; therefore, 2 is the logarithm of 9 to base 3, or \(2 =log_{3} 9\)

Now, we will use the laws of the logarithm to expand the term log (ab)^2 as follows -

log \((ab)^{2}\)

= 2 log (ab)

Using the product property, we will get,

= 2(log a + log b)      

= 2log a + 2log b

Hence, the expanded form of log \((ab)^{2}\) is 2log a + 2log b.

Read more about logarithms:

brainly.com/question/10208274

#SPJ4

The complete question is -

Use the laws of the logarithm to expand the term log (ab)^2.

Which real-life situation involves a linear function whose graph passes through the points? Y (0, 20) and x (4, 80)
A) the entrance fee to an amusement park is $15 and each book of ride tickets costs $20.
B) Jet ski rental costs $20 initially plus $15 for each hour.
C) Shoes are on a sale at a shoe store for $20 for the first pair and $17 for each additional pair.
D) You earn and deposit into a bank account money from chores at the rate of $20 per week.

Answers

Answer:

B) Jet ski rental costs $20 initially plus $15 for each hour.

Step-by-step explanation:

The equation of a line has the following format:

\(y = mx+b\)

In which m is the slope, which having two points on the line, is given by the change in y divided by the change in x, and b is the y-intercept, which is the value of y when \(x = 0\).

Passes through point Y (0, 20):

This means that \(b = 20\). So

\(y = mx+20\)

Slope:

Through points (0,20) and (4,80). So

Change in y: 80 - 20 = 60

Change in x: 4 - 0 = 4

Slope: 60/4 = 15

So

\(y = 15x+20\)

Situation:

We want a situation in which we have a fixed fee of 20, and then for each unit of time or of the product, and increase of 15. So the correct option is B

Fiona enlarges a parallelogram that has a perimeter of 17 meters.

A small parallelogram has a left side length of 3.5 meters. A larger parallelogram has a left side length of 21 meters.
Not drawn to scale

What is the perimeter of the enlarged parallelogram?
30 meters
73.5 meters
102 meters
105 meters

Answers

Answer:

The correct answer is C.

Step-by-step explanation:

Answer: C. 102 cm

Step-by-step explanation:

I took the quiz on Edge, and it is correct.

Help me answer this.

Help me answer this.

Answers

iven:

here are givren the expression:

\((2z-y)^{20}\)

xplanation:

o find the coefficient of the given expression, we need to use the binomial expansion formula:

So,

From the formula of binomial expansion:

\((x+y)^n=\sum_{k\mathop{=}0}^n(n,k)x^ky^{n-k}\)

then,

After using the above formula in the given binomial, the expansion will be:

\((2z-y)^{20}=y^{20}-40y^{19}z+760y^{18}z^2-9120y^{17}z^3+77520y^{16}z^4-496128y^{15}z^5+...\)

inal answer:

ence , the coefficient of the given term is shown below:

\(77520\)

Find the future value of an annuity due with a monthly payment of $300 at 6% interest compounded
monthly for 10 years.
O $4117.46
O $49163.80
O $49409.62
$52113.63
O $52374.20
O $109,163.80

Answers

Answer: The future value of an annuity due with a monthly payment of $300 at 6% interest compounded monthly for 10 years can be calculated using the formula:

FV = PMT * ( ( (1 + r)^n - 1) / r) * (1 + r)

where PMT = $300, r = 6%/12 = 0.05, and n = 10 years * 12 months/year = 120 months.

Substituting these values into the formula, we get:

FV = $300 * ( ( (1 + 0.05)^120 - 1) / 0.05) * (1 + 0.05)

FV = $300 * ( ( 1.05^120 - 1) / 0.05) * 1.05

FV = $300 * ( ( 2.704813829421526 - 1) / 0.05) * 1.05

FV = $300 * ( 1.704813829421526 / 0.05) * 1.05

FV = $300 * 34.09627658843153 * 1.05

FV = $52,113.63

Therefore, the future value of the annuity due is $52,113.63.

Step-by-step explanation:

If y varies directly as x, and y = 5 when x = 4, find y when x = 8.

Answers

Since y is directly proportional to x and y = 5 when x = 4, the value of y is equal to 10.

What is a direct variation?

In Mathematics, a direct variation can be modeled by this mathematical expression:

y = kx

Where:

x and y represents the variables or data points.k represents the constant of proportionality.

Based on the information provided above, we would determine the constant of proportionality (k) by substituting the value of the given variable as follows:

k = y/x

k = 5/4

Now, we can determine the value of y:

y = kx

y = 5/4 × 8

y = 10.

Read more on variation here: https://brainly.com/question/16455054

#SPJ1


What is the value of c?
a)4 units
b)5 units
c)6 units
d)7 units

What is the value of c?a)4 unitsb)5 unitsc)6 unitsd)7 units

Answers

The value of c in the triangle is (b) 5 units

Finding the value of c in the triangle

From the question, we have the following parameters that can be used in our computation:

The right triangle

The length c is the hypotenuse of one of the triangles and can be calculated using the following Pythagoras theorem

c² = sum of squares of the legs

Using the above as a guide, we have the following:

c² = 3² + 4²

Evaluate

c² = 25

Take the square roots

c = 5

Hence, the hypotenuse of the right triangle is 5

Read more about right triangle at

https://brainly.com/question/2437195

#SPJ1

Vedant received two quotes for new carpet for his home. The quote from Company A is $0.15 less per square foot than the quote from Company B. If Vedant plans to buy 1,200 square feet of carpet, how do his total costs from the two companies compare?
Company A is cheaper by $180.

Company B is cheaper by $180.

Company A is cheaper by $8,000.

Company B is cheaper by $8,000.

Don’t make any funny jokes about this and don’t say idk or something like that or I’ll report you.

Answers

Answer:

Company B is cheaper by $180.

Step-by-step explanation:

just do 1,200 square feet x 0.15 = 180

Answer:

b as in big cat

Step-by-step explanation:

Passes through the points (-2,-9) and (5,-8). Find the y-intercept

Answers

Answer:

slope  1/7

y intercept: (0, -67)

slope intercept form: y=1/7x - 61/7

Step-by-step explanation:

point slope form is

y+9=1/7 *(x+2)

The dental fees for a root canal treatment vary from one office to another. According to a survey conducted by an insurance company, the distribution of the fees is normal with the mean of $1,100 and the SD of $50. To compare the fees in Riverside County to the nation as a whole, we took a sample of eight dental offices in the county. What is the sampling distribution of the average cost of a root canal treatment in Riverside County?

Answers

Answer:

The  sampling distribution of the average cost of a root canal treatment in Riverside County is normal  with a mean of  $1,100  and  standard deviation of \(\sigma_{\= x} = \$ 17.67\)

Step-by-step explanation:

From the question we are told that

    The mean is \(\mu = \$ 1,100\)

      The standard deviation is  \(\sigma = \$ 50\)

     The  sample size is  n =  8

Generally the standard deviation of the mean of the sampling distribution is mathematically represented as

            \(\sigma_{\= x} = \frac{\sigma }{\sqrt{n} }\)

=>        \(\sigma_{\= x} = \frac{50 }{\sqrt{8} }\)

=>        \(\sigma_{\= x} = \$ 17.67\)

Generally given that the sampling distribution of the fees in the nation as a whole is normal with a with the mean of $1,100  and  the SD of $50

Then the   \(\= x\)( sample mean of the country side ) is  normally distributed with a mean of  $1,100  and  standard deviation of \(\sigma_{\= x} = \$ 17.67\)

which is the equation of a line that has a slope of -2/3 and passes through point (-3, -1)?

Answers

Answer: y = (-2/3)x - 3

Step-by-step explanation:

We'll be using y = mx + b

m = slope

b = y-intercept

Because we know this line will pass through -3, -1, we can input that.

y = mx + b

-1 = (-2/3)(-3) + b

-1 = 2 + b

b = -3

We can input that now to get the equation

y = (-2/3)x - 3

please I need the answer right now.

please I need the answer right now.

Answers

3/10, this shouls be correct

Find the measure of the third interior angle of a triangle, given the measure of two angles.

62 and 24 and ______

Answers

We know that the sum of the internal angles is equal to 180°, then we write and solve a linear equation to see that the missing angle is 94°.

How to find the measure of the third angle?

Remember that for any triangle, we know that the sum of the interior angles is equal to 180°.

In this case we know that two of the angles have a measure of 62° and 24°, and if the third angle is x, then we can write the linear equation:

62° + 24° + x = 180°

To solve this we need to isolate x, we will get:

x = 180° - 24° - 62°

x = 94°

So the measure of the third interior angle is 94 degrees.

Learn more about interior angles by reading:

https://brainly.com/question/25215131

#SP1

The following probability distribution represents the payout for a certain dice game

x P(x)
−5 0.17
−3 0.21
−1 0.09
3 0.23
5 0.23
7 0.07


a.) What is the expected payout?



b.) Is it expected that a person playing this particular game will win money or lose money?

win money since the expected value is positive
lose money since the expected value is negative


c.) What is the standard deviation (round to four decimal places)?

Answers

The standard deviation by the given  probability distribution is 3.3412.

What do you mean by standard deviation?

In statistics, Standard deviation is a measure of the variation of a set of values.

σ = standard deviation of population

N = number of observation of population

X = mean

μ = population mean

We are given that;

x P(x)

−5 0.17

−3 0.21

−1 0.09

3 0.23

5 0.23

7 0.07

Now,

To find the standard deviation, we need to use the formula:

standard deviation = sqrt[∑(x - μ)²P(x)]

where μ is the expected payout.

We have already calculated μ to be 0.76. Using this value and the given probabilities, we can calculate the standard deviation as:

standard deviation = sqrt[( (-5 - 0.76)² * 0.17) + ((-3 - 0.76)² * 0.21) + ((-1 - 0.76)² * 0.09) + ((3 - 0.76)² * 0.23) + ((5 - 0.76)² * 0.23) + ((7 - 0.76)² * 0.07)]

=3.3412

Therefore, by the given data the standard deviation will be 3.3412.

Learn more about standard deviation and variance:

https://brainly.com/question/11448982

#SPJ1

The midpoint of \overline{\text{AB}} AB is M(7, -7)M(7,−7). If the coordinates of AA are (8, -6)(8,−6), what are the coordinates of BB?

Answers

Using the formula for the midpoint of the line with given points  A and B as the end points of the line, the coordinates of the point B is (6,-8).

How do you find the midpoint of a line segment?

To find the midpoint of a line segment, you must first find the coordinates of the two endpoints of the segment. Then, take the average of the x-coordinates and the y-coordinates to find the x and y coordinates of the midpoint. This can be done by adding the x-coordinates and y-coordinates of the two endpoints together and then dividing by 2.

What is the formula for finding the midpoint of a line segment?

The formula for finding the midpoint of a line segment is:

Midpoint = ( (x1+x2)/2 , (y1+y2)/2 )

Where (x1,y1) and (x2,y2) are the coordinates of the two endpoints of the line segment.

Using the given coordinates of point A(8,-6) and the midpoint M(7,-7) of the line,  and using the midpoint formula for a line AB,

Midpoint(M) = \((\frac{x1+x2}{2},\frac{y1+y2}{2} )\)

where x1, y1 are coordinates of point A , and x2, y2 are the coordinates of point B

so , (7,-7) = \((\frac{8+x2}{2} , \frac{-6+y2}{2})\)

equating the corresponding points,

7 =(8+x2)/2

therefore, x2 = 6

and , -7 = (-6+y2)/2

therefore , y2 = -8

Hence the coordinates of B is , (x2,y2 ) = (6,-8)

To learn more about midpoint of a line visit:

https://brainly.com/question/28224145

#SPJ1

help with 6 please i need to graduate

help with 6 please i need to graduate

Answers

The expression with the correct coefficients in each block is given as follows:

20x³ + 0x² - 7x.

How to obtain the resultant function?

The functions in the context of this problem are defined as follows:

f(x) = 4x.g(x) = 5x² - 4.h(x) = 9x.

The operation is given as follows:

f(x)g(x) + h(x).

Hence, to obtain the resultant function, we apply the definitions into the operation and the simplify, applying the distributive property and combining the like terms, as follows:

4x(5x² - 4) + 9x = 20x³ - 16x + 9x = 20x³ - 7x.

As there is no x² term, the coefficient of zero multiplies the term x².

More can be learned about functions at https://brainly.com/question/31895757

#SPJ1

Express as a difference: a+5

Answers

Ax5 a➗5 and a-5

................

The expression expressed as a difference is (a - 2) - (-7).

We have,

We can express a + 5 as a difference by subtracting a suitable term from it.

One way to do this is to subtract 2 and add 2, like this:

a + 5 = a + 5 - 2 + 2   (adding and subtracting 2)

     = (a - 2) + 7     (rearranging)

Now,

We can express a + 5 as the difference between a - 2 and -7, like this:

a + 5 = (a - 2) - (-7)

Thus,

The expression expressed as a difference is (a - 2) - (-7).

Learn more about expressions here:

https://brainly.com/question/30968842

#SPJ2

At a baseball game a vendor sold a combined total of 136 sodas and hotdogs. the number of soda sold was three times the number of the hotdogs sold. find the number of sodas in the number of hotdogs sold

Answers

Answer:

Step-by-step explanation:

Let's call the number of hotdogs sold "x". According to the problem, the number of sodas sold is three times the number of hotdogs sold, or 3x.

We know that the total number of sodas and hotdogs sold is 136. So we can set up an equation:

x + 3x = 136

Simplifying this equation, we get:

4x = 136

Dividing both sides by 4, we get:

x = 34

This means that 34 hotdogs were sold. To find the number of sodas sold, we can multiply the number of hotdogs sold by 3:

3x = 3(34) = 102

So, 102 sodas were sold.

i’ll give brainliest

ill give brainliest

Answers

Answer:

Verticle stretch

Step-by-step explanation:

it is vetically stretched by 2.

Which of the expressions below are equal to 5x + 20y? Select all that apply.
A) 5(x + 4y)
B) 3x + 12y + 2x + 8y
C) 3x + 10y + 3x + 8y
D) 2x + 15y + 2x + 5y


Help plzzz

Which of the expressions below are equal to 5x + 20y? Select all that apply.A) 5(x + 4y)B) 3x + 12y +

Answers

The correct answers are A and B.
Hope this helps!
A & B are the correct answers

Determine if the sequence below is arithmetic or geometric and determine the common difference / ratio in simplest form. 4, 14, 24,

Answers

Answer:

It is the arithmetic sequence and the common ratio is 10

Step-by-step explanation:

It is arithmetic sequence

 This is because

-Geometric is multiplication

-arithmetic is addition  

----

Now the common ratio/difference is 10 as it adds 10 each time

4, 4+10 (14), 14+10 (24)

A deep sea diver descended from a rock that is 45 feet below

Answers

Diver descended 43 feet from the rock to the coral reef.
A deep sea diver descended from a rock that is 45 feet below sea level to a coral reef that is 88 feet below sea level.
To find : How far did the diver descend from the rock to the coral reef.
Solution : We have given
A deep sea diver descended below sea level = -45 feet .
A coral reef that is 88 feet below sea level = -88 feet.
Because we take the negative value below sea level( 0 ).
Difference = -45 - ( - 88)
Difference = -45 + 88 = 43
Diver descended 43 feet from the rock to the coral reef.
Therefore, Diver descended 43 feet from the rock to the coral reef.
If that is your question...

-4 is greater than of equal to x - 11

Answers

Using the inequality rule we know that 11 is greater than -11, (11 > -11).

What is an Inequality Equation?

Mathematical expressions with inequalities are those in which the two sides are not equal.

Contrary to equations, we compare two values in inequality.

Less than (or less than or equal to), greater than (or greater than or equal to), or not equal to signs are used in place of the equal sign.

Two expressions in inequality aren't always equal, which is denoted by the symbols >, <, ≤, or ≥.

So, let's say the inequality equation looks like this: A

Right now, A is valued at

Let p be used to representing the initial number.

P equals 11, so

Let q be used to representing the second number.

Q has a value of -11.

-11 and 11 are both numbers that are greater than zero.

Then, 11 > -11 ... (1)

A has a value of 11 > -11.

Therefore, using the inequality rule we know that 11 is greater than -11, (11 > -11).

Know more about Inequality here:

brainly.com/question/11897796

#SPJ1

Correct question:
Is 11 greater than, equal to, or less than -11?

The effectiveness of a blood-pressure drug is being investigated. An experimenter finds that, on average, the reduction in systolic blood pressure is 23.5 for a sample of size 775 and standard deviation 12.2. Estimate how much the drug will lower a typical patient's systolic blood pressure (using a 95% confidence level). Enter your answer as a tri-linear inequality accurate to one decimal place (because the sample statistics are reported accurate to one decimal place).

Answers

The 95% confidence interval for the effectiveness of the blood-pressure drug is given as follows:

\(22.6 < \mu < 24.4\)

How to obtain the confidence interval?

The mean, the standard deviation and the sample size for this problem, which are the three parameters, are given as follows:

\(\overline{x} = 23.5, \sigma = 12.2, n = 775\)

Looking at the z-table, the critical value for a 95% confidence interval is given as follows:

z = 1.96.

The lower bound of the interval is then given as follows:

\(23.5 - 1.96 \times \frac{12.2}{\sqrt{775}} = 22.6\)

The upper bound of the interval is then given as follows:

\(23.5 + 1.96 \times \frac{12.2}{\sqrt{775}} = 24.4\)

More can be learned about the z-distribution at https://brainly.com/question/25890103

#SPJ1

Name the image of p (9, 1.5) after being translated along the vector <3, -0.5>

Answers

The image for P(9, 1.5) after Translation is P'( 12, 1).

What is Translation?

Each point in a figure is moved the same distance in the same direction using a type of transformation known as translation.

For example, this transformation moves the parallelogram to the right 5 units and up 3 units. It is written ( x , y ) → ( x + 5 , y + 3 ) .

Given:

P(9, 1.5) with Translating Vector (3, -0.5).

So, Applying the Translation we get

P(9 + 3, 1.5 + (-0.5))

= P(12 , 1.5- 0.5)

= P(12, 1)

hence, the image is P'( 12, 1).

Learn more about translation here:

https://brainly.com/question/12463306

#SPJ1

A to City B. In 5 ​days, they have traveled 2,075 miles. At this​ rate, how long will it take them to travel from City A to City​ B?

Answers

In the question, we can draw the conclusion that, according to the formula, it will take them 10 days to get from City A to City B if they continue travelling at their current average speed of \(415 miles\)per day.

What is formula?

A formula is a set of mathematical signs and figures that demonstrate how to solve a problem.

Formulas for calculating the volume of \(3D\) objects and formulas for measuring the perimeter and area of \(2D\) shapes are two examples.

A formula is a fact or a rule in mathematical symbols. In most cases, an equal sign connects two or more values. If you know the value of one, you can use a formula to calculate the value of another quantity.

We need to know the average pace at which they went to figure how long it would take to get from City A to City B at the same rate.

total distance / time taken = average speed

\(415 miles\) per day \(2075/ 5\), it would take them \(10\) days to get from City A to City B because

Time taken = \(2075/415\) per days \(= 5 days\)

Therefore it will take them 10 days to get from City A to City B if they continue travelling at their current average speed of \(415 miles\)per day.

Learn more about speed

https://brainly.com/question/28224010

#SPJ1

Need help don’t know what to do I’m confused

Need help dont know what to do Im confused

Answers

Im pretty sure it’s D. Y= 0.5x + 3 :)

According to the 2018 general social survey (gss), 60% of all u. s. households have at least one pet. a veterinarian in ann arbor decided to conduct a survey to assess if over 60% of all ann arbor households have at least one pet. a 10% significance level was selected to be used. the random sample of 280 ann arbor households, resulted in 182 stating they currently have at least one pet. these results are turned over to you to perform the appropriate test. 1) state the appropriate hypotheses to be tested with a complete definition of the parameter of interest. (the significance level has been selected to be 10%). 2) check the necessary assumption(s) (you may assume that the selected sample is a random sample). 3) perform the appropriate test (include all supporting computations for the test statistic and p-value) 4) give your decision and provide a conclusion in context. your answer needs to be organized well and use labels for your steps 1, 2, 3, and 4. question 8 answer no answer entered.

Answers

Step-by-step explanation:

1. the claim here is that over 60 percents of the households living in ann harbor have pets.

null hypothesis;

h0: p≤0.60

alternative:

h1: p>0.60

2.

n is the size of the sample = 280

n(p) = 280 x 0.60 = 168 greater than 10

n(1-p) = 280(1-0.6) = 280 x 0.4 = 112  greater than 10

we use the normal dist since each of the above are greater than 10

3. n = 280

x = 182

x/n = 182/280

= 0.65

we get the test statistic z

z = \(\frac{0.65-0.6}{\sqrt{0.6*(1-0.6)/280} }\)

= \(\frac{0.05}{\sqrt{0.000857} }\)

=\(\frac{0.05}{0.02927}\)

≈ \(1.71\)

we get the p value = 1-p(z<1.71)

= 1-.9562

this gives 0.0438

4. we can see that 0.0438 < 0.10

so we conclude that over 60% of these households have at least one pet since we rejected h0.

Other Questions
Provide the quantum numbers and related information for the following orbital, 5s. a) What is the principal quantum number, n? b) What is the angular momentum quantum number, I? c) What is the number of degenerate orbitals based on the magnetic quantum number?d) How many radial nodes are there? I have to follow this direction to get the answer. I'm totally stuck What does it mean if a galaxy gives off light that has been shifted toward the red end of the electromagnetic spectrum? aretha franklin wrote her hit "respect" before soul artist otis redding changed it into a mens rights anthem.T/F Solve.7x+5-3=0Select the correct choice below and fill in any answer boxes present in your choice.OA. The solution(s) is(are) x =(Type an integer or a simplified fraction. Use a comma to separate answers as needed.)OB. There is no solution. A student is given the rectangle and the square shown. The student states that the two figures have thesame perimeter. Is the student correct? Complete the explanation to explain your reasoning.X + 10X +5X +5The student is (select) The perimeter of a figure is found by (select)For the rectangle, the perimeter isFor the square, the perimeterthe lengths of its sides,is On June 1, 2019, Splish Company sold $3,720,000 in long-term bonds for $3,262,800. The bonds will mature in 10 years and have a stated interest rate of 8% and a yield rate of 10%. The bonds pay interest annually on May 31 of each year. The bonds are to be accounted for under the effective-interest method.Required:Construct a bond amortization table for this problem to indicate the amount of interest expense and discount amortization at each May 31. At times a buyers voices opinions or concerns more to vent frustration than anything else It allow us to be more effective in customer care. Bookwork code: L19Calculatornot allowedb) Find the value of w.Give each answer as an integer or as a fraction in its simplest form.task4 cmA7 cm12 cm3 cmWatch videoBw cm9 cmNot drawn accurately How is the young man's choice between the "lady" or the "tiger" decided? question 2 options: by the king by chance by the princess by the reader Anne, the HR director, has a complicated message to deliver in person to one of her employees. The message will also require backup written communication. She needs to ensure that she and the employee reach a common understanding on an important matter. Anne's message has information richness ambiguity urgency time sensitivity What is the product? 2x+8 x2 + 3x-4 x(x-4)(x-1) 20x+4) 2 (x+4)(x-4) 2x(x-1) 2x(x+4) True or False, according to fiedler's contingency theory, a leader's style does not change; therefore, if there is a mismatch between a situation and the leader who is in place, it is best to move that leader to another position. prior to administering morphine sulfate to a client in the postanesthesia recover unit (pacu), what information must the nurse obtain? (select all that apply.) Calculate the external self-inductance of the coaxial cable in the previous question if the space between the line conductor and the outer conductor is made of an inhomogeneous material having = 2( 2(1-p) Hint: Flux method might be easier to get the answer. Acetic acid has a Ka of 1.75x10^-5. The weak acid approximation (for this class) would be allowed for which of the following initial concentrations of acetic acid?a. 1.75E^-6 b. 1.75E^-5 c. 0.00175 d. 0.0175 e. 0.175 True/False: Write A if you believe the statement is correct and B if you believe thestatement is incorrect. (36-44) How do I find all horizontal and vertical asymptotes of a curve and justify my answers with limit statements? 50 points!!!!!!!Jeremy has heard a lot of people in the club talking about tackling a 950-foot rock formation. Jeremy has set a limit that they wont climb anything that takes longer than 7 hours, so they have enough time to travel to and from the place. Will it be possible to climb a 950-foot rock formation within Jeremys time constraint? Explain why or why not.